Rotating the roots of a cubic

Algebra Level 5

Consider all pairs of polynomials ( f ( x ) , g ( x ) ) (f(x), g(x)) with complex coefficients such that

  1. f ( x ) f(x) is a monic polynomial of degree 3 and has distinct roots α 1 , α 2 , α 3 \alpha_1, \alpha_2, \alpha_3 .
  2. g ( x ) g(x) is a linear polynomial such that g ( α 1 ) = α 2 , g ( α 2 ) = α 3 , g ( α 3 ) = α 1 . g(\alpha_1)=\alpha_2,\ g(\alpha_2)=\alpha_3,\ g(\alpha_3)=\alpha_1.
  3. f ( 0 ) = 2 f(0) = 2 .
  4. f ( 1 ) = 9 f(1) = 9 .

The sum of all possible (distinct) values of f ( 2 ) f(2) is N N . What are the last three digits of N N ?

Details and assumptions

A polynomial is monic if its leading coefficient is 1. For example, the polynomial x 3 + 3 x 5 x^3 + 3x - 5 is monic but the polynomial x 4 + 2 x 3 6 -x^4 + 2x^3 - 6 is not.


The answer is 38.

This section requires Javascript.
You are seeing this because something didn't load right. We suggest you, (a) try refreshing the page, (b) enabling javascript if it is disabled on your browser and, finally, (c) loading the non-javascript version of this page . We're sorry about the hassle.

11 solutions

Jatin Yadav
Aug 27, 2013

Let f ( x ) = x 3 + a x 2 + b x + c , f ( 0 ) = c = 2 , f ( 1 ) = 1 + a + b + c = 3 + a + b = 9 a + b = 6. f(x) = x^3 + ax^2 + bx + c , f(0) = c = 2 , \\ f(1) = 1 + a + b + c = 3 + a + b = 9 \Rightarrow a + b = 6. Now , Let g ( x ) = p x + q . g(x) = px + q . g ( α 1 ) = p α 1 + q = α 2 . . . . . ( i ) , g ( α 2 ) = p α 2 + q = α 3 . . . . . . . ( i i ) , a n d g ( α 3 ) = p α 3 + q = α 1 . . . . . . ( i i i ) . g(\alpha_{1}) = p\alpha_{1} + q = \alpha_{2}.....(i), \\ g(\alpha_{2}) = p\alpha_{2} + q = \alpha_{3}.......(ii) , and \\ g(\alpha_{3}) = p\alpha_{3} + q = \alpha_{1}......(iii). Using ( i ) (i) and ( i i ) (ii) , we get p = α 2 α 3 α 1 α 2 \frac{\alpha_{2} - \alpha_{3}}{\alpha_{1} - \alpha_{2}} , Using ( i i ) (ii) and ( i i i ) (iii) , we get p = α 3 α 1 α 2 α 3 \frac{\alpha_{3} - \alpha_{1}}{\alpha_{2} - \alpha_{3}} . Equating the two , ( α 1 ) 2 + ( α 2 ) 2 + ( α 3 ) 2 = α 1 α 2 + α 2 α 3 + α 3 α 1 ( α 1 + α 2 + α 3 ) 2 = 3 ( α 1 α 2 + α 2 α 3 + α 3 α 1 ) a 2 = 3 b (\alpha_{1})^2 + (\alpha_{2})^2 + (\alpha_{3})^2 = \alpha_{1}\alpha_{2} + \alpha_{2}\alpha_{3} + \alpha_{3}\alpha_{1} \\ \Rightarrow (\alpha_{1} + \alpha_{2} + \alpha_{3})^2 = 3( \alpha_{1}\alpha_{2} + \alpha_{2}\alpha_{3} + \alpha_{3}\alpha_{1}) \Rightarrow a^2 = 3b , also, b = 6 a a 2 = 3 ( 6 a ) a 2 + 3 a 18 = 0 a = 3 b = 6 - a \Rightarrow a^2 = 3( 6 - a) \Rightarrow a^2 + 3a - 18 = 0 \Rightarrow \fbox{ a = 3} or a = -6 \fbox{ a = -6} Correspondingly , b = 3 \fbox{ b = 3 } , or b = 12 \fbox{b = 12} Put these values to get , f ( x ) = x 3 + 3 x 2 + 3 x + 2 f(x) = x^3 + 3x^2 + 3x + 2 or f ( x ) = x 3 6 x 2 + 12 x + 2 f ( 2 ) = 28 f(x) = x^3 - 6x^2 + 12x +2 \Rightarrow f(2) = 28 or f ( 2 ) = 10. f(2) = 10. Hence, sum of possible values = 28 + 10 = 38 \fbox{38}

Moderator note:

Nicely done!

Well explained solution. I did this in a similar way.

Vali Dobre - 7 years, 9 months ago

Very clever use of manipulation of algebra and vieta's formulas. Nice job Jatin Y. (btw, your formatting kind of got screwed up on line 6)

Abhishek Saraiya - 7 years, 9 months ago

Log in to reply

can you please tell how to break the lines , the always appear together , even though have been broken .

jatin yadav - 7 years, 9 months ago

Log in to reply

Put extra blank lines between them

Matt McNabb - 7 years, 9 months ago

Log in to reply

@Matt McNabb thanks!

However , i just got it yesterday.

jatin yadav - 7 years, 9 months ago

Brilliant solution. Though I have a slight contention.

When you derive relationship between alpha's in line 8, that can be written as,

(1/2) * [ (a1 - a2)^2 + (a2 - a3)^2 + (a3 - a1)^2 ] = 0, which gives a1 = a2 = a3 = 0, thus contradicting distinct root assumption.

I know this has no bearing on the problem, yet something that struck me as contradictory.

(Note: a1, a2, a3 represent three alpha's. Pardon my formatting, I'm not good at this.)

Mirza Baig - 7 years, 9 months ago

Log in to reply

An equation can be written in a number of ways. Keeping in mind, the data available to you, you should focus on attaining the ONE, you can solve. Your solution leads to nowhere. That's why try a different arrangement of variables, to lead to a feasible solution, that's what he did.

Arnab Animesh Das - 7 years, 9 months ago

actually, this is only one equation , we have to solve this equation with α 1 + α 2 + α 3 = a , α 1 α 2 α 3 = 2 , α 1 α 2 + α 2 α 3 + α 3 α 1 = b , a + b = 6 \alpha_1 + \alpha_2 + \alpha_3 = -a , \alpha_1 \alpha_2 \alpha_3 = 2 , \alpha_1 \alpha_2 + \alpha_2 \alpha_3 + \alpha_3 \alpha_1 = b , a + b = 6 , but α 1 = α 2 = α 3 = 0 \alpha_1 = \alpha_2 = \alpha_3 = 0 won't satisfy these equations.

jatin yadav - 7 years, 9 months ago

In Complex number set, a^2+b^2+c^2=0 does not implies that a=0 and b=0 and c=0. We have a great application here :) ... but a^2+b^2=0 involves a=0 and b=0, in C? (if no, can we give an example?)

Virgilius Teodorescu - 7 years, 9 months ago

Log in to reply

a 2 + b 2 = 0 a^2+b^2 = 0 does not necessarily imply a = 0 a=0 and b = 0 b=0 . For example, take a = 1 + i a=1+i and b = 1 i b=1-i . Then a 2 = 2 i a^2=2i and b 2 = 2 i b^2=-2i , so a 2 + b 2 = 0 a^2+b^2=0

Nhat Le - 7 years, 9 months ago

Yesterday, I solved it. But the term "complex coefficients" was driving me mad. So today, I gave in, and gave it a try, due to the available no. of submissions, and Voila!! LOL

Arnab Animesh Das - 7 years, 9 months ago

Log in to reply

Note that real is a subset of complex.

Taehyung Kim - 7 years, 9 months ago

Furthermore, α 1 2 + α 2 2 + α 3 2 = α 1 α 2 + α 2 α 3 + α 3 α 1 \alpha_1^2+\alpha_2^2+\alpha_3^2=\alpha_1\alpha_2+\alpha_2\alpha_3+\alpha_3\alpha_1 is the condition for α 1 , α 2 , α 3 \alpha_1,\alpha_2,\alpha_3 being the vertices of an equilateral triangle in the complex plane!

(see my solution for the proof)

Steven Hao - 7 years, 9 months ago

Very nice!

My own line of thinking got me stuck, but I don't know why; Function g g satisfies g 3 ( x ) = x g^3(x)=x , from this I thought g g had to be a rotation of order 3 3 . I find it hard to imagine a linear function on the complex plane that is not a rotation but does satisfy the given.

Ton de Moree - 7 years, 9 months ago

Log in to reply

Your intuition is correct; in general, a linear function with complex coefficients is a rotation and dilation about some point in the complex plane.

Steven Hao - 7 years, 9 months ago

Log in to reply

Alright, the problem arose when I put g ( x ) = x e 2 π i 3 g(x)=xe^\frac{2\pi i}{3} . I figured that the radius of the rotation wouldn't change due to the fact that in three steps you'd have to be back at α 1 \alpha_1 . Eventually this led me to α 1 = α 2 = α 3 = 0 \alpha_1=\alpha_2=\alpha_3=0

Ton de Moree - 7 years, 9 months ago
Mark Hennings
Aug 26, 2013

Suppose that g ( x ) = a x + b g(x)=ax+b . Then α 2 = a α 1 + b α 3 = a α 2 + b = a 2 α 1 + ( a + 1 ) b α 1 = a α 3 + b = a 3 α 1 + ( a 2 + a + 1 ) b \begin{array}{rcl} \alpha_2 & = & a\alpha_1 + b \\ \alpha_3 & = & a\alpha_2 + b \; = \; a^2\alpha_1 + (a+1)b \\ \alpha_1 & = & a\alpha_3 + b \; = \; a^3\alpha_1 + (a^2+a+1)b \end{array} and so ( a 2 + a + 1 ) [ ( a 1 ) α 1 + b ] = ( a 2 + a + 1 ) ( α 2 α 1 ) = 0 (a^2+a+1)\big[(a-1)\alpha_1 + b\big] \; = \; (a^2 + a + 1)(\alpha_2-\alpha_1) \; = \; 0 Since we could replace g g by g 1 g^{-1} , we can assume without loss of generality that a = ω a = \omega , the principal primitive cube root of unity.

Introducing c = b 1 ω c = \tfrac{b}{1-\omega} , we note that f ( c ) = c f(c)=c . We can write α 1 = c + d α 2 = c + ω d α 3 = x + ω 2 d \alpha_1 \; = \; c + d \qquad \alpha_2 \; = \; c + \omega d \qquad \alpha_3 \ = \; x + \omega^2d where d 0 d \neq 0 . Thus f ( x ) = ( x c ) 3 d 3 f(x) \,=\, (x-c)^3 - d^3 . Since c 3 d 3 = f ( 0 ) = 2 ( 1 c ) 3 d 3 = f ( 1 ) = 9 -c^3 - d^3 \; = \; f(0) \; = \; 2 \qquad \qquad (1-c)^3 - d^3 \; = \; f(1) \; = \; 9 we deduce that c 3 + ( 1 c ) 3 = 7 c^3 + (1-c)^3 \,=\, 7 , or c 2 c 2 = 0 c^2 - c - 2 \,=\, 0 , so that c = 2 , 1 c = 2,-1 .

  • If c = 2 c=2 then d 3 = 10 d^3 = -10 , so that f ( x ) = ( x 2 ) 3 + 10 f(x) = (x-2)^3 + 10 . Thus f ( 2 ) = 10 f(2) = 10 .

  • If c = 1 c=-1 then d 3 = 1 d^3 = -1 , so that f ( x ) = ( x + 1 ) 3 + 1 f(x) = (x+1)^3 + 1 . Thus f ( 2 ) = 28 f(2) = 28 .

Adding these answers together, we deduce that N = 38 N = 38 .

Moderator note:

Great job! This is basically what we had in mind in this problem.

Since we could replace g g by g 1 g^{-1} , we can assume without loss of generality that a = ω a= \omega ,the principal primitive cube root of unity.

Can you please explain this step? Also, how can we introduce c = b 1 a c= \frac{b}{1-a} , WLOG? Aren't we taking particular values of the coefficients?

Sreejato Bhattacharya - 7 years, 9 months ago

Log in to reply

Since α 1 α 2 \alpha_1 \neq \alpha_2 , we must have a 2 + a + 1 = 0 a^2+a+1=0 , and hence a a is either ω \omega or ω 2 \omega^2 . Since a 1 a \neq 1 , the function g ( x ) g(x) is invertible. We could use g 1 ( x ) g^{-1}(x) instead of g ( x ) g(x) without altering the argument ( g 1 g^{-1} would just cycle through the roots of f ( x ) f(x) in the opposite order). If a = ω 2 a=\omega^2 then the coefficient of x x in g 1 ( x ) g^{-1}(x) would be ω \omega . Thus we just choose whichever of g g and g 1 g^{-1} has ω \omega as the coefficient of x x , and decide to call that function g ( x ) g(x) .

As to c c , I am noting that the function g ( x ) g(x) has a fixed point, and that g ( x ) g(x) is basically a rotation through 12 0 120^\circ about the fixed point c c . This becomes clear once you express x x in terms of its displacement from c c . Obviously, the value of c c depends on the value of b b . The point of doing this is to make the forms of α 1 , α 2 , α 3 \alpha_1,\alpha_2,\alpha_3 very pretty, so that the form of f ( x ) = ( x c ) 3 d 3 f(x) = (x-c)^3 - d^3 is neat. This approach is completely general.

Mark Hennings - 7 years, 9 months ago

If you imagine looking at the logic for the solution in the complex plane, you could flip the whole thing about the x-axis and get back the same solution (because w 2 w^2 is the complex conjugate of w w ).

Matt McNabb - 7 years, 9 months ago

g(x) is linear. How come we are getting its coefficients as complex numbers ?

Mirza Baig - 7 years, 9 months ago

Log in to reply

There is nothing that says that a linear polynomial must have real coefficients, and the question specifically permits them to be complex. Moreover, although the conditions force f ( x ) f(x) to have real coefficients, there is no g ( x ) g(x) with real coefficients which satisfies the conditions. Look at the identity ( a 2 + a + 1 ) ( α 1 α 2 ) = 0 (a^2+a+1)(\alpha_1-\alpha_2) = 0 If a a were real, then a 2 + a + 1 > 0 a^2+a+1 > 0 , which would force α 1 = α 2 \alpha_1=\alpha_2 . But we are told that the roots of f ( x ) f(x) are distinct.

Mark Hennings - 7 years, 9 months ago

"linear" means degree 1 (doesn't say anything about the coefficients except that the first coefficient is not zero).

Matt McNabb - 7 years, 9 months ago

Same thing with me, but I guess Jatin Y. made this problem a simple business.

A Brilliant Member - 7 years, 9 months ago

What prompted you to introduce c and d? I did it without those and the algebra was quite complicated, but I eventually got solutions 28 28 and 66 66 . I guess I had a small error somewhere, but didn't have time to go over everything again after that stupid sign error on the f(g) problem :)

Matt McNabb - 7 years, 9 months ago

Log in to reply

I wanted to show that g ( x ) g(x) was a rotation - with ω \omega as the coefficient of x x it had to be. Consider the title of the problem.

Mark Hennings - 7 years, 9 months ago
Nhat Le
Aug 27, 2013

From the first condition, we deduce that f ( x ) = ( x α 1 ) ( x α 2 ) ( x α 3 ) f(x)= (x-\alpha_1)(x-\alpha_2)(x-\alpha_3)

Since f ( 0 ) = 2 f(0)=2 , we have ( α 1 ) ( α 2 ) ( α 3 ) = 2 (-\alpha_1)(-\alpha_2)(-\alpha_3)=2 , or α 1 α 2 α 3 = 2 \alpha_1 \alpha_2 \alpha_3 =-2

Since f ( 1 ) = 9 f(1)=9 , we have ( 1 α 1 ) ( 1 α 2 ) ( 1 α 3 ) = 9 (1-\alpha_1)(1-\alpha_2)(1-\alpha_3)=9 . Expanding the brackets gives us 1 ( α 1 + α 2 + α 3 ) + ( α 1 α 2 + α 2 α 3 + α 1 α 3 ) α 1 α 2 α 3 = 9 1-(\alpha_1+\alpha_2+\alpha_3)+(\alpha_1\alpha_2+\alpha_2\alpha_3+\alpha_1\alpha_3)-\alpha_1 \alpha_2 \alpha_3=9 , and since we know that α 1 α 2 α 3 = 2 \alpha_1 \alpha_2 \alpha_3=-2 , this simplifies to ( α 1 + α 2 + α 3 ) + ( α 1 α 2 + α 2 α 3 + α 1 α 3 ) = 6 ( ) -(\alpha_1+\alpha_2+\alpha_3)+(\alpha_1\alpha_2+\alpha_2\alpha_3+\alpha_1\alpha_3) = 6 (*)

Now we focus on the second condition. Let g ( x ) = m x + b g(x)=mx+b , then we have m α 1 + b = α 2 m\alpha_1+b=\alpha_2 , m α 2 + b = α 3 m\alpha_2+b=\alpha_3 , and m α 3 + b = α 1 m\alpha_3+b=\alpha_1 . Subtracting the first equation from the second equation gives us m ( α 2 α 1 ) = α 3 α 2 m(\alpha_2-\alpha_1)=\alpha_3-\alpha_2 and subtracting the second equation from the third equation gives us m ( α 3 α 2 ) = α 1 α 3 m(\alpha_3-\alpha_2)=\alpha_1-\alpha_3 .

Since α 1 , α 2 , α 3 \alpha_1,\alpha_2,\alpha_3 are distinct, we thus have m = α 3 α 2 α 2 α 1 = α 1 α 3 α 3 α 2 m=\frac{\alpha_3-\alpha_2}{\alpha_2-\alpha_1}=\frac{\alpha_1-\alpha_3}{\alpha_3-\alpha_2}

Cross-multiplying, we have ( α 3 α 2 ) 2 = ( α 2 α 1 ) ( α 1 α 3 ) (\alpha_3-\alpha_2)^2=(\alpha_2-\alpha_1)(\alpha_1-\alpha_3) , and after simplification, this gives us α 1 2 + α 2 2 + α 3 2 = α 1 α 2 + α 2 α 3 + α 3 α 1 \alpha_1^2+\alpha_2^2+\alpha_3^2=\alpha_1\alpha_2+\alpha_2\alpha_3+\alpha_3\alpha_1 , which is equivalent to ( α 1 + α 2 + α 3 ) 2 = 3 ( α 1 α 2 + α 2 α 3 + α 3 α 1 ) (\alpha_1+\alpha_2+\alpha_3)^2=3(\alpha_1\alpha_2+\alpha_2\alpha_3+\alpha_3\alpha_1)

If we let A = α 1 + α 2 + α 3 A=\alpha_1+\alpha_2+\alpha_3 and B = α 1 α 2 + α 2 α 3 + α 3 α 1 B=\alpha_1\alpha_2+\alpha_2\alpha_3+\alpha_3\alpha_1 , we would have A 2 = 3 B A^2=3B . Furthermore from equation (*), B = A + 6 B=A+6 . Thus, A 2 = 3 ( A + 6 ) A^2=3(A+6) , implying A 2 3 A 18 = 0 A^2-3A-18=0 , which means A = 6 A=6 or A = 3 A=-3

Therefore we have two cases: A = 6 , B = 12 A=6,B=12 or A = 3 , B = 3 A=-3,B=3 . Using Vieta's theorem: f ( x ) = x 3 ( α 1 + α 2 + α 3 ) x 2 + ( α 1 α 2 + α 2 α 3 + α 3 α 1 ) x α 1 α 2 α 3 f(x)=x^3-(\alpha_1+\alpha_2+\alpha_3)x^2+(\alpha_1\alpha_2+\alpha_2\alpha_3+\alpha_3\alpha_1)x-\alpha_1\alpha_2\alpha_3 , we arrive at two possible functions:

f ( x ) = x 3 6 x 2 + 12 x + 2 f(x)=x^3-6x^2+12x+2 or f ( x ) = x 3 + 3 x 2 + 3 x + 2 f(x)=x^3+3x^2+3x+2

In the first case, f ( 2 ) = 10 f(2)=10 and in the second case, f ( 2 ) = 28 f(2) = 28 . Thus the sum of the possible values of f ( 2 ) f(2) is 10 + 28 = 38 10+28=\fbox{38}

Moderator note:

Nicely done!

A polynomial f f has the form f ( z ) = z 3 a z 2 + b z c , f(z) = z^3 - az^2 + b z - c, where a = α 1 + α 2 + α 3 , b = α 1 α 2 + α 2 α 3 + α 3 α 1 , a = \alpha_1 + \alpha_2 + \alpha_3,\, b = \alpha_1\alpha_2+\alpha_2\alpha_3 + \alpha_3\alpha_1, and c = α 1 α 2 α 3 . c=\alpha_1\alpha_2\alpha_3. From conditions 3 and 4, we have c = 2 and b a = 6. ( ) c=-2\quad \text{and}\quad b-a= 6.\quad\quad(*) Let g ( z ) = α z + β . g(z) = \alpha z + \beta. Condition 2 implies g 3 ( α i ) = g ( g ( g ( α i ) ) ) = α i for i = 1 , 2 , 3. g^3(\alpha_i)=g(g(g(\alpha_i))) = \alpha_i\quad \text{for}\,\, \, i=1,2,3. Together with the fact that g 3 ( z ) = α 3 z + β ( α 2 + α + 1 ) g^3(z) = \alpha^3 z + \beta(\alpha^2+\alpha+1) we have α 3 α i + β ( α 2 + α + 1 ) = α i for i = 1 , 2 , 3. \alpha^3 \alpha_i + \beta(\alpha^2+\alpha+1) = \alpha_i \quad \text{for} \,\,\, i=1,2,3. Subtracting the above equations with i = 1 i =1 and i = 2 i=2 we have α 3 ( α 1 α 2 ) = α 1 α 2 . \alpha^3(\alpha_1 - \alpha_2) = \alpha_1-\alpha_2. By the assumption that α 1 α 2 \alpha_1 \neq \alpha_2 we have α 3 = 1. \alpha^3 = 1. This implies α \alpha is a cube root of unity. If α = 1 , \alpha =1, then one of the above equations yields α 1 + 3 β = α 1 , \alpha_1 + 3\beta = \alpha_1, so that β = 0 \beta = 0 and we have g ( z ) = α z + β = z g(z) = \alpha z + \beta = z implying α 2 = g ( α 1 ) = α 1 , \alpha_2 = g(\alpha_1) = \alpha_1, a contradiction. Hence α 1 1. \alpha_1 \neq 1. This implies α \alpha is a primitive cube root of unity. Let's denote it by ω . \omega. Hence g ( z ) = ω z + β g(z) = \omega z + \beta or β = g ( z ) ω z . \beta = g(z) -\omega z. Substituting z = α i z=\alpha_i for i = 1 , 2 , 3 i=1,2,3 into this equation, we have α 2 ω α 1 = α 3 ω α 2 = α 1 ω α 3 . \alpha_2-\omega \alpha_1 = \alpha_3-\omega \alpha_2 = \alpha_1 - \omega \alpha_3. The first pair of the equations yields ( 1 + ω ) α 2 = ω α 1 + α 3 (1+\omega)\alpha_2 = \omega \alpha_1+\alpha_3 or ω α 1 + ω 2 α 2 + α 3 = 0 , \omega \alpha_1 +\omega^2 \alpha_2 + \alpha_3 = 0, since ω 2 + ω + 1 = 0. \omega^2+\omega+1 = 0. We have similar relations with the other two pairs of the equations. We can translate these relations into a matrix equation as follows: A X = 0 , AX = \mathbf{0}, where A = ( α 1 α 2 α 3 α 2 α 3 α 1 α 3 α 1 α 2 ) A=\begin{pmatrix} \alpha_1 & \alpha_2 & \alpha_3 \\ \alpha_2 & \alpha_3 & \alpha_1\\ \alpha_3 & \alpha_1 & \alpha_2\end{pmatrix} and X = ( 1 ω ω 2 ) . X= \begin{pmatrix} 1 \\ \omega \\ \omega^2 \end{pmatrix}. Since X 0 , X\neq\mathbf{0}, this implies det A = 0. \det A = 0. (Otherwise A A is invertible yielding X = A 1 0 = 0 , X = A^{-1}\mathbf{0} = \mathbf{0}, a contradiction.) Adding the second and third rows to the first row, and factoring, we have det A = ( α 1 + α 2 + α 3 ) 1 1 1 α 2 α 3 α 1 α 3 α 1 α 2 . \det A = (\alpha_1+\alpha_2+\alpha_3)\begin{vmatrix} 1 & 1 & 1 \\ \alpha_2 & \alpha_3 & \alpha_1 \\ \alpha_3 & \alpha_1 & \alpha_2 \end{vmatrix}. Expanding along the first row, we obtain det A = c y c α 1 ( c y c α 1 α 2 c y c α 1 2 ) . \det A = \sum_{\mathrm{cyc}}\alpha_1\Big(\sum_{\mathrm{cyc}}\alpha_1\alpha_2 - \sum_{\mathrm{cyc}}\alpha_1^2\Big). Imposing det A = 0 \det A = 0 yields either terms of the product to be 0 0 . If c y c α 1 = 0 , \sum_{\mathrm{cyc}}\alpha_1 = 0, then subtracting from the relation ω α 1 + ω 2 α 2 + α 3 = 0 \omega\alpha_1 +\omega^2\alpha_2 +\alpha_3 = 0 yields α 1 + ( ω + 1 ) α 2 = 0 \alpha_1 + (\omega+1)\alpha_2 = 0 or α 1 = ω 2 α 2 \alpha_1 = \omega^2 \alpha_2 or ω α 1 = α 2 , \omega \alpha_1 = \alpha_2, where the last two equations follow from the facts that ω 2 + ω + 1 = 0 \omega^2 + \omega + 1 = 0 and ω 2 = 1 ω . \omega^2 = \frac{1}{\omega}. Similarly, we obtain ω α 2 = α 3 \omega \alpha_2 = \alpha_3 and ω α 3 = α 1 . \omega \alpha_3 = \alpha_1. Multiplying these three equations by α 3 , α 1 , \alpha_3,\alpha_1, and α 2 , \alpha_2, respectively and combining, we have ω c y c α 1 α 2 = c y c α 1 α 2 . \omega\sum_{\mathrm{cyc}}\alpha_1\alpha_2 = \sum_{\mathrm{cyc}}\alpha_1\alpha_2. Since ω 0 , \omega \neq 0, we have c y c α 1 α 2 = 0. \sum_{\mathrm{cyc}}\alpha_1\alpha_2 =0. In conclusion, we have a = c y c α 1 = 0 and b = c y c α 1 α 2 = 0 , a=\sum_{\mathrm{cyc}}\alpha_1 = 0 \quad \text{and} \quad b = \sum_{\mathrm{cyc}}\alpha_1\alpha_2 = 0, contradicting the equation ( ) (*) above.

Therefore c y c α 1 α 2 c y c α 1 2 = 0. \sum_{\mathrm{cyc}}\alpha_1\alpha_2 - \sum_{\mathrm{cyc}}\alpha_1^2 = 0. In terms of a a and b , b, this is 3 b a 2 = 0. 3b-a^2 = 0. Combining with the equation ( ) (*) we have ( a , b ) = ( 6 , 12 ) , ( 3 , 3 ) . (a,b) = (6,12),\,(-3,3). With c = 2 c=-2 , this yields two possible polynomials f , f, namely f ( z ) = z 3 6 z 2 + 12 z + 2 and f ( z ) = z 3 + 3 z 2 + 3 z + 2. f(z) =z^3-6z^2+12z+2 \quad \text{and} \quad f(z) =z^3+3z^2+3z+2. This gives two possible values of f ( 2 ) f(2) . They are f ( 2 ) = 2 3 6 ( 2 2 ) + 12 ( 2 ) + 2 = 10 f(2) = 2^3-6(2^2)+12(2)+2 = 10 and f ( 2 ) = 2 3 + 3 ( 2 2 ) + 3 ( 2 ) + 2 = 28. f(2) = 2^3+3(2^2)+3(2)+2 = 28. Hence the sum of all possible values of f ( 2 ) f(2) is 10 + 28 = 38 . 10+28 = \boxed{38}.

Nice solution. Bu one has to know some linear algebra to appreciate it.

Alexander Borisov - 7 years, 9 months ago
Alija Bevrnja
Aug 26, 2013

This is not the way I came to the solution, but these steps are a bit more motivated.

Using 1. condition ( f(x) is monic) , we can write:

f(x) = (x - α1)(x - α2)(x - α3).

From 3. and 4. we have:

-α1 α2 α3 = 2 ...(#) and

(1 - α1)(1 - α2)(1 - α3) = 9 which can be written as:

1 - (α1 + α2 + α3) + (α1 α2 + α1 α3 + α2 α3) - α1 α2 α3 = 9,

using (#) we get

-(α1 + α2 + α3) + (α1 α2 + α1 α3 + α2 α3) = 6 (##)

We are asked for f(2):

f(2) = (2 - α1)(2 - α2)(2 - α3)

f(2) = 8 - 4(α1 + α2 + α3) + 2(α1 α2 + α1 α3 + α2 α3) - α1 α2 α3,

and using (#)

f(2) = 10 - 4(α1 + α2 + α3) + 2(α1 α2 + α1 α3 + α2 α3)

Here we realize that we only need to find (α1 + α2 + α3) and

(α1 α2 + α1 α3 + α2 α3) if possible.

Now we use condition 2. writing down the g(x):

g(x) = Ax + B, so

A α1 + B = α2 ...(1)

A α2 + B = α3 ...(2)

A α3 + B = α1 ...(3)

We do not need A and B, so we try to eliminate them, first A then B.

Multiplying (1) with α2 and (2) with α1 and taking difference (and extracting B):

B (α2 - α1) = α2^2 - α3 α1

Doing the same with (2) and (3) and also with (3) and (1), always eliminating A we get:

B (α3 - α2) = α3^2 - α1 α2

B (α1 - α3) = α1^2 - α2 α3

Summing these 3 equations and rearranging we get:

α1^2 + α2^2 + α3^2 = α1 α2 + α2 α3 + α3 α1 ... (###)

Now we can write a known relation:

(α1 + α2 + α3)^2 = α1^2 + α2^2 + α3^2 + 2(α1 α2 + α2 α3 + α3 α1),

using (###)

(α1 + α2 + α3)^2 = 3(α1 α2 + α2 α3 + α3 α1).

Expressing (α1 α2 + α2 α3 + α3 α1) from (##) and putting here we get:

(α1 + α2 + α3)^2 = 3(6 + (α1 + α2 + α3))

Marking the sum (α1 + α2 + α3) with k we obtain:

k^2 - 3k - 18 = 0, and therefore:

k1 = -3 and k2 = 6

From (##) we find (α1 α2 + α1 α3 + α2 α3) and put into f(2) together with k:

a) k = k1 = -3

f(2) = 28

b) k = k2 = 6

f(2) = 10

Therefore, sum of all f(2) is 38. Ta da! :D

1 upvote for the concluding sentence

Nhat Le - 7 years, 9 months ago
C Lim
Aug 25, 2013

Step 1 : Restrict possibilities for f ( x ) f(x) .

By conditions 3 and 4, f ( x ) ( 7 x + 2 ) f(x) - (7x+2) vanishes at x = 0 , 1 x=0, 1 so it is a multiple of x ( x 1 ) x(x-1) . Write f ( x ) = h ( x ) x ( x 1 ) + 7 x + 2 f(x) = h(x)x(x-1) + 7x+2 for some linear polynomial h ( x ) h(x) . Since f ( x ) f(x) is monic, so is h ( x ) h(x) . So f ( x ) = ( x a ) x ( x 1 ) + 7 x + 2 f(x) = (x-a)x(x-1) + 7x+2 for some complex a a .

Step 2 : Use the second condition to deduce properties of α i \alpha_i .

Write g ( x ) = b x + c g(x) = bx + c . Then g ( x 1 ) g ( x 2 ) = b ( x 1 x 2 ) g(x_1) - g(x_2) = b(x_1 - x_2) . Hence:

α 3 α 2 = g ( α 2 ) g ( α 1 ) = b ( α 2 α 1 ) . \alpha_3 - \alpha_2 = g(\alpha_2) - g(\alpha_1) = b(\alpha_2 - \alpha_1).

By the same token, α 2 α 1 = b ( α 1 α 3 ) \alpha_2 - \alpha_1 = b(\alpha_1 - \alpha_3) and α 1 α 3 = b ( α 3 α 2 ) \alpha_1 - \alpha_3 = b(\alpha_3 - \alpha_2) so we have: b 3 ( α 3 α 2 ) = 1 b^3(\alpha_3 - \alpha_2) = 1 . If b 3 0 b^3 \ne 0 , then all the alpha's are equal, which is impossible by condition 1. Hence b 3 = 1 b^3 = 1 so b = 1 b=1 or b b is a third root of unity.

In the first case, b = 1 b=1 implies α 2 α 1 = α 1 α 3 \alpha_2 - \alpha_1 = \alpha_1 - \alpha_3 and so α 1 + α 2 + α 3 = 3 α 1 \alpha_1 + \alpha_2 + \alpha_3 = 3\alpha_1 . By symmetry, this value also equals 3 α 2 3\alpha_2 and 3 α 3 3\alpha_3 and so all alphas are equal, which is impossible. Hence: b = ω b = \omega is a third root of unity. Using the equality ω 2 + ω + 1 = 0 \omega^2 + \omega + 1 = 0 , we get:

α 2 α 1 = ω ( α 1 α 3 ) , α 3 α 2 = ω 2 ( α 1 α 3 ) . \alpha_2 - \alpha_1 = \omega(\alpha_1 - \alpha_3),\ \alpha_3 - \alpha_2 = \omega^2(\alpha_1 - \alpha_3).

This gives:

( α 2 α 1 ) 2 + ( α 1 α 3 ) 2 + ( α 3 α 2 ) 2 = ( 1 + ω 2 + ω 4 ) ( α 1 α 3 ) 2 = 0. (\alpha_2 - \alpha_1)^2 + (\alpha_1 - \alpha_3)^2 + (\alpha_3 - \alpha_2)^2 = (1+\omega^2 + \omega^4)(\alpha_1 - \alpha_3)^2 = 0.

Hence: α 1 2 + α 2 2 + α 3 2 = α 1 α 2 + α 2 α 3 + α 3 α 1 . \alpha_1^2 + \alpha_2^2 + \alpha_3^2 = \alpha_1 \alpha_2 + \alpha_2 \alpha_3 + \alpha_3 \alpha_1.

Step 3 : Solve for a a .

We have: f ( x ) = x 3 ( a + 1 ) x 2 + ( 7 + a ) x + 2 f(x) = x^3 - (a+1)x^2 + (7+a)x + 2 so the roots satisfy:

α 1 + α 2 + α 3 = a + 1 , α 1 α 2 + α 2 α 3 + α 3 α 1 = a + 7. \alpha_1 + \alpha_2 + \alpha_3 = a+1, \ \alpha_1 \alpha_2 + \alpha_2 \alpha_3 + \alpha_3 \alpha_1 = a+7.

From the conclusion at step 2, we get: ( a + 1 ) 2 = 3 ( a + 7 ) (a+1)^2 = 3(a+7) which gives a 2 a 20 = 0 a^2 - a - 20 = 0 . Hence, a = 5 , 4 a= 5, -4 , which corresponds to f ( 2 ) = 28 , 10 f(2) = 28, 10 respectively.

[ Note : one also needs a certain amount of work to show that all these three steps are reversible, i.e. that a = 5 , 4 a=5, -4 are both valid solutions. ]

Not Understand Your Answer, Correct Answer Is ?

Jhonny Means - 7 years, 9 months ago
Peter Fedak
Aug 28, 2013

Suppose g ( x ) = m x + b g(x)=mx+b . Note that g ( g ( g ( α i ) ) ) = α i g(g(g(\alpha_i)))=\alpha_i for i = 1 , 2 , 3 i=1,2,3 . In general, we have g ( g ( g ( x ) ) ) = m 3 x + m 2 b + m b + b g(g(g(x))) = m^3 x+m^2b+mb+b . Then we have α i ( 1 m 3 ) = m 2 b + m b + b \alpha_i(1-m^3) = m^2b+mb+b , so if m 3 1 m^3\neq 1 we would have all roots identical, equal to b 1 m \frac{b}{1-m} . Then it must be that m 3 = 1 m^3=1 . m = 1 m=1 forces b = 0 b=0 , again making roots identical, so we must have m m a primitive third root of unity.

Suppose r r is a root of f f . Then the other roots are m r + b mr+b and m 2 r + m b + b = m 2 r m 2 b m^2r+mb+b = m^2r-m^2b , and so f ( x ) = ( x r ) ( x m 2 r + m 2 b ) ( x m r b ) f(x) = (x-r)(x-m^2 r+m^2 b)(x-m r-b) . Note that if we had instead picked g ( x ) = m 2 x m 2 b g(x)=m^2 x-m^2 b , the inverse of our current choice, we would end up with the same set of roots for f f , and so the set of polynomials f f doesn't depend on which of the two third roots of unity m m is. Expanding, noting that f ( 0 ) = 2 f(0)=2 fixes the constant term as 2, we have f ( x ) = x 3 ( m 2 r + m r + r + b m 2 b ) x 2 + ( m 3 r 2 + m 2 r 2 + m r 2 + r b m 2 r b m 3 r b + m 2 r b m 2 b 2 ) x + 2 f(x) = x^3 -(m^2r+mr+r+b-m^2b)x^2 +(m^3r^2+m^2r^2+mr^2+rb-m^2 rb-m^3 rb+m^2 rb-m^2b^2)x+2 . Or, simplifying, f ( x ) = x 3 + ( m 2 1 ) b x 2 m 2 b 2 x + 2 f(x)= x^3+(m^2-1)bx^2-m^2b^2x+2 , where we have used m 3 = 1 m^3=1 and m 2 + m + 1 = 0 m^2+m+1=0 .

Now, enforcing f ( 1 ) = 9 f(1)=9 gives 1 + m 2 b b m 2 b 2 + 2 = 9 1+m^2b-b-m^2b^2+2=9 , or m 2 b b m 2 b 2 = 6 m^2b-b-m^2b^2=6 . We have f ( 2 ) = 8 + 4 ( m 2 b b ) 2 m 2 b 2 + 2 f(2)=8+4(m^2b-b)-2m^2b^2+2 . Substituting in the previous expression gives f ( 2 ) = 10 + 2 6 + 2 ( m 2 1 ) b f(2) = 10+2\cdot 6+2(m^2-1)b . We note that the f ( 1 ) = 0 f(1)=0 condition determines b b as the root of a quadratic, and the value of f ( 2 ) f(2) depends only on the value of b b . Then the answer is 22 + ( m 2 1 ) b 1 + 22 + ( m 2 1 ) b 2 22+(m^2-1)b_1+22+(m^2-1)b_2 for b i b_i the possible values of b. The sum of values of b b is m 2 1 m 2 = 1 m \frac{m^2-1}{m^2} = 1-m by Vieta's formula. Then our answer is 44 + 2 ( m 2 1 ) ( 1 m ) = 44 + 2 ( m 2 + m 2 ) = 44 6 = 38 44+2(m^2-1)(1-m) = 44+2(m^2+m-2) = 44-6 = 38 .

Ganesh Sundaram
Aug 27, 2013

First condition implies f ( x ) = ( x α 1 ) ( x α 2 ) ( x α 3 ) . f(x) = (x-\alpha_1)(x-\alpha_2)(x-\alpha_3). The condition 3 implies f ( 0 ) = α 1 α 2 α 3 = 2. ( # 1 ) f(0) = - \alpha_1 \alpha_2 \alpha_3 = 2. \qquad (\#1) The condition 4 implies, after using Eq.(#1), α 1 α 2 α i = 6 , ( # 2 ) \sum \alpha_1 \alpha_2 - \sum \alpha_i = 6, \qquad (\#2) where α 1 α 2 α 1 α 2 + α 2 α 3 + α 3 α 1 \sum \alpha_1\alpha_2 \equiv \alpha_1\alpha_2 + \alpha_2\alpha_3 + \alpha_3\alpha_1 .

The condition 2 implies that g ( x ) = a x + b , and g(x) = ax + b, \quad \text{and} a α 1 + b = α 2 ; a α 2 + b = α 3 ; a α 3 + b = α 1 . a \, \alpha_1 + b = \alpha_2; \quad a \, \alpha_2 + b = \alpha_3; \quad a \, \alpha_3 + b = \alpha_1. Solving for a a with the first two equations and the last two equation and matching them, after some algebra, gives α i 2 = α 1 α 2 \sum \alpha_i^2 = \sum \alpha_1 \alpha_2 \quad \Rightarrow ( α i ) 2 = α i 2 + 2 α 1 α 2 = 3 α 1 α 2 . \left( \sum \alpha_i \right)^2 = \sum \alpha_i^2 + 2\sum \alpha_1 \alpha_2 = 3\sum \alpha_1 \alpha_2. The same equation is obtained by doing the same for b b as well. Substituting this in equation (#2) gives, ( α i ) 2 3 α i 18 = 0. \left( \sum \alpha_i \right)^2 - 3\sum \alpha_i - 18 = 0. Solving the above equation we find that the possibilities are α i = 6 , α 1 α 2 = 12 ; \sum \alpha_i = 6, \quad \sum \alpha_1 \alpha_2 = 12; and α i = 3 , α 1 α 2 = 3. \sum \alpha_i = -3, \quad \sum \alpha_1 \alpha_2 = 3. Now for f ( 2 ) f(2) , after using Eq.(#1), f ( 2 ) = 10 + 2 α 1 α 2 4 α i . f(2) = 10 + 2 \sum \alpha_1 \alpha_2 - 4 \sum \alpha_i. Thus, the possible values of f ( 2 ) f(2) are 10 and 28, whose sum is 38.

Let f ( x ) = x 3 + a x 2 + b x + c f(x)= x^3 + ax^2 + bx + c . Since f ( 0 ) = 2 f(0)= 2 , c = 2 c= 2 . Since f ( 1 ) = 9 f(1)= 9 , a + b + c + 1 = 9 a + b = 6 a+b+c+1= 9 \implies a+b= 6 . Hence, we have f ( x ) = x 3 + a x 2 + ( 6 a ) x + 2 f(x)= x^3 + ax^2 + (6-a)x + 2 for some a a . We wish to determine all possible values for a a , as different values for a a produce different possibilities for f ( x ) f(x) .

According to Vieta's formula, we have the following relations. α 1 + α 2 + α 3 = a \alpha_1 + \alpha_2 + \alpha_3= -a α 1 α 2 + α 2 α 3 + α 3 α 1 = 6 a \alpha_1 \alpha_2 + \alpha_2 \alpha_3 + \alpha_3 \alpha_1 = 6-a α 1 α 2 α 3 = 2 \alpha_1 \alpha_2 \alpha_3 = -2

Now let g ( x ) = p x + q g(x)= px+q , where p , q p,q are constants and p 0 p \neq 0 . According to the question, we have the following relations.
p α 1 + q = α 2 . . . . . ( 1 ) p \alpha_1 + q= \alpha_2 \ .....(1) p α 2 + q = α 3 . . . . . ( 2 ) p \alpha_2 + q= \alpha_3 \ .....(2) p α 3 + q = α 1 . . . . . ( 3 ) p \alpha_3 + q= \alpha_1 \ .....(3)

Solving for p p and q q in equations ( 1 ) (1) and ( 2 ) (2) yield
p = α 2 α 3 α 1 α 2 p= \frac{\alpha_2 - \alpha_3}{\alpha_1-\alpha_2} q = α 1 α 3 α 2 2 α 1 α 2 q= \frac{\alpha_1 \alpha_3 - \alpha_2^2}{\alpha_1-\alpha_2}

Plugging these values in ( 3 ) (3) and multiplying both sides by α 1 α 2 \alpha_1 - \alpha_2 , we obtain α 1 α 3 α 2 2 + α 2 α 3 α 3 2 = α 1 2 α 2 α 2 \alpha_1 \alpha_3 - \alpha_2^2 + \alpha_2 \alpha_3 - \alpha_3^2= \alpha_1^2 - \alpha_2 \alpha_2 α 1 α 2 + α 2 α 3 + α 3 α 1 = α 1 2 + α 2 2 + α 3 2 \implies \alpha_1 \alpha_2 + \alpha_2 \alpha_3 + \alpha_3 \alpha_1 = \alpha_1^2 + \alpha_2^2 + \alpha_3 ^2

But we know that α 1 α 2 + α 2 α 3 + α 3 α 1 = 6 a \alpha_1 \alpha_2 + \alpha_2 \alpha_3 + \alpha_3 \alpha_1 = 6-a α 1 2 + α 2 2 + α 3 2 = ( α 1 + α 2 + α 3 ) 2 2 ( α 1 α 2 + α 2 α 3 + α 3 α 1 ) = a 2 2 ( 6 a ) \alpha_1 ^2 + \alpha_2^2 + \alpha_3^2 = (\alpha_1 + \alpha_2 + \alpha_3)^2 - 2(\alpha_1 \alpha_2 + \alpha_2 \alpha_3 + \alpha_3 \alpha_1) = a^2 - 2(6-a) .

Hence, we find out 6 a = a 2 2 ( 6 a ) 6-a = a^2 - 2(6-a) . Solving yields a = 6 , + 3 a= -6, +3 , which gives the corresponding possibilities for f ( x ) f(x) : f ( x ) = x 3 6 x 2 + 12 x + 2 f(x)= x^3 - 6x^2 + 12x + 2 o r or f ( x ) = x 3 + 3 x 2 + 3 x + 2 f(x)= x^3 + 3x^2 + 3x + 2

Thus, the sum of all possible values of f ( 2 ) f(2) is 2 3 6. 2 2 + 12.2 + 2 + 2 3 + 3. 2 2 + 3.2 + 2 = 38 2^3 - 6.2^2 + 12.2 + 2 + 2^3 + 3.2^2 + 3.2 + 2 = \boxed{38} .

This solution is slightly incomplete, as it does not provide justification why the coefficients of g ( x ) g(x) are defined. Luckily though, by taking derivatives it can be checked that none of the polynomials mentioned have repeated roots.

Sreejato Bhattacharya - 7 years, 9 months ago
Steven Hao
Aug 31, 2013

Because f f is monic, f ( x ) = ( x α 1 ) ( x α 2 ) ( x α 3 ) f(x) = (x-\alpha-1)(x-\alpha_2)(x-\alpha_3) Let A = c y c α 1 , B = c y c α 1 α 2 , C = α 1 α 2 α 3 \displaystyle{A=\sum_{cyc}\alpha_1,\\B=\sum_{cyc}\alpha_1\alpha_2,\\C=\alpha_1\alpha_2\alpha_3} Substituting f ( 0 ) , f ( 1 ) f(0), f(1) gives us the following. f ( 0 ) = C = 2 C = 2 f(0)=-C=2\implies C=-2 f ( 1 ) = 1 A + B C = 9 B = A + 6 f(1)=1-A+B-C=9\implies B=A+6

To use the "cyclic root" condition, note that g g , a linear function, is an affine transformation; it rotates, dilates, and translates. Specifically, it has a constant angle of rotation. Since g g takes the line segment α 1 α 2 \alpha_1\alpha_2 to α 2 α 3 \alpha_2\alpha_3 , its angle of rotation is equal to the exterior angle at α 2 \alpha_2 . By symmetry, that angle of rotation is also equal to the exterior angles at α 3 , α 1 \alpha_3, \alpha_1 . With 3 equal angles, the triangle formed by α 1 , α 2 , α 3 \alpha_1,\alpha_2,\alpha_3 is equilateral.

Lemma: For all equilateral triangles with vertices at complex numbers x , y , z x, y, z , we have the following identity: x 2 + y 2 + z 2 = x y + y z + z x x^2+y^2+z^2=xy+yz+zx Proof: ( z x ) = ± ω ( y x ) (z-x)=\pm\omega (y-x) where ω \omega is a primitive sixth root of unity. Specifically, ω 2 ω + 1 = 0 \omega^2-\omega+1=0 Then ( z x y x ) 2 ( z x y x ) + 1 = 0 ( z x ) 2 ( z x ) ( y x ) + ( y x ) 2 = 0 x 2 + y 2 + z 2 = x y + y z + z x \left(\frac{z-x}{y-x}\right)^2-\left(\frac{z-x}{y-x}\right)+1=0\\ (z-x)^2-(z-x)(y-x)+(y-x)^2=0\\ x^2+y^2+z^2=xy+yz+zx

Using this identity, we can plug in this "cyclic roots" condition easily. c y c α 1 2 = B A 2 2 B = B A 2 = 3 B \displaystyle{\sum_{cyc}\alpha_1^2=B\\ A^2-2B=B\\ A^2=3B}

We substitute B = A + 6 B=A+6 to obtain a quadratic in A A . A 2 = 3 ( A + 6 ) A 2 3 A 18 = 0 A^2=3(A+6)\\ A^2-3A-18=0 There are two roots for A A , and their sum is 3 3 .

We wish to find the sum of possible values of f ( 2 ) f(2) . f ( 2 ) = 8 4 A + 2 B C = 8 4 A + 2 ( A + 6 ) ( 2 ) = 22 2 A f(2)=8-4A+2B-C=8-4A+2(A+6)-(-2)=22-2A The sum of the possible values of f ( 2 ) f(2) (there are 2 of them) is then simply 22 2 2 ( 3 ) = 38 22\cdot2 - 2(3) = \boxed{38} .

i made in a typo in the first line: replace α 1 \alpha-1 with α 1 \alpha_1 also note that, in contrast with other solutions, there is very little computation in this solution :)

Steven Hao - 7 years, 9 months ago
Tran Dinh Duy Vu
Aug 27, 2013

-from the condition of g(x) we can obtain that \frac{\alpha {3}-\alpha {2}}{\alpha {2}-\alpha {1}} = \frac{\alpha {1}-\alpha {3}}{\alpha {3}-\alpha {2}} \Rightarrow \alpha {1}^2 + \alpha{2}^2 + \alpha{3}^2 = \alpha {1} \alpha {2} + \alpha {2} \alpha {3} + \alpha {1} \alpha {3} Add 2(\alpha {1} \alpha {2} + \alpha {2} \alpha {3} + \alpha {1} \alpha {3} ) to both sides we obtain (\alpha {1}+\alpha {2}+\alpha {3})^2 = 3(\alpha {1} \alpha {2} + \alpha {2} \alpha {3} + \alpha {1} \alpha {3} ) -from the condition of the problem, f(x) = (x-\alpha {1})(x-\alpha {2})(x-\alpha {3}) f(0) = 2 then \alpha {1} \alpha {2} \alpha {3} = -2 f(1) = 9 then 1 -(\alpha {1}+\alpha {2}+\alpha {3}) + (\alpha {1} \alpha {2} + \alpha {2} \alpha {3} + \alpha {1} \alpha {3} ) - \alpha {1} \alpha {2} \alpha {3} = 9 \rightarrow \frac{(\alpha {1}+\alpha {2}+\alpha {3})^2}{3} - (\alpha {1}+\alpha {2}+\alpha {3}) -6 =0 then (\alpha {1}+\alpha {2}+\alpha {3}) = -3 or 6 f(2) = 8 - 4(\alpha {1}+\alpha {2}+\alpha {3}) + \frac{2}{3}(\alpha {1}+\alpha {2}+\alpha_{3})^2 +2 Therefore the answer is 16 - 4(6-3) + \frac{2}{3}(6^2 +3^2) + 4 = 38

0 pending reports

×

Problem Loading...

Note Loading...

Set Loading...